Why didn't they simplify this?












8












$begingroup$


Solving $x^y = y^x$ analytically in terms of the Lambert $W$ function



This "solution" for $x^y=y^x$ should simplify to $y=x$, but for some reason no pointed that out in the OP.



According to the stack exchange, the answer is $y= frac{-xW(-frac{ln(x)}{x})}{ln(x)}$. However, the term $frac{-ln(x)}{x}$ itself can be rewritten as



$$frac{-ln(x)}{x}=-ln(x)e^{-ln(x)}$$



Therefore, the productlog of that expression should simplify as follows,



$y= frac{-xW(-frac{ln(x)}{x})}{ln(x)}, $ $y= frac{-xW(-ln(x)e^{-ln(x)})}{ln(x)}, $ $y=frac{-x(-ln(x))}{ln(x)}=x$



Did this simplification just slip past everyone or is there something wrong about my algebra?










share|cite|improve this question











$endgroup$








  • 4




    $begingroup$
    Why should it reduce to that? $x=4$ and $y=2$ has $x neq y$.
    $endgroup$
    – Randall
    9 hours ago






  • 2




    $begingroup$
    No, $2^4=16=4^2$.
    $endgroup$
    – Randall
    9 hours ago






  • 5




    $begingroup$
    I'm just confused why the solution "should" simplify to $x=y$ when there are solutions that do not satisfy $x = y$.
    $endgroup$
    – Randall
    9 hours ago






  • 1




    $begingroup$
    Anyway, to potentially answer your question, your algebra moves are invalid if $x$ is negative, and there are solutions with negative $x$.
    $endgroup$
    – Randall
    9 hours ago






  • 1




    $begingroup$
    As motivation, you might consider whether the relation $y = arcsin(sin(x))$ is "algebraically equivalent" to $y=x$. (What about when $y=0$ and $x=pi$?) The same issue is at play here: $v = W(u)$ means that $u e^u = v$, but there is not always a unique $u$ with that property, and depending on which one you choose, your claim may or may not be valid.
    $endgroup$
    – Nate Eldredge
    8 hours ago
















8












$begingroup$


Solving $x^y = y^x$ analytically in terms of the Lambert $W$ function



This "solution" for $x^y=y^x$ should simplify to $y=x$, but for some reason no pointed that out in the OP.



According to the stack exchange, the answer is $y= frac{-xW(-frac{ln(x)}{x})}{ln(x)}$. However, the term $frac{-ln(x)}{x}$ itself can be rewritten as



$$frac{-ln(x)}{x}=-ln(x)e^{-ln(x)}$$



Therefore, the productlog of that expression should simplify as follows,



$y= frac{-xW(-frac{ln(x)}{x})}{ln(x)}, $ $y= frac{-xW(-ln(x)e^{-ln(x)})}{ln(x)}, $ $y=frac{-x(-ln(x))}{ln(x)}=x$



Did this simplification just slip past everyone or is there something wrong about my algebra?










share|cite|improve this question











$endgroup$








  • 4




    $begingroup$
    Why should it reduce to that? $x=4$ and $y=2$ has $x neq y$.
    $endgroup$
    – Randall
    9 hours ago






  • 2




    $begingroup$
    No, $2^4=16=4^2$.
    $endgroup$
    – Randall
    9 hours ago






  • 5




    $begingroup$
    I'm just confused why the solution "should" simplify to $x=y$ when there are solutions that do not satisfy $x = y$.
    $endgroup$
    – Randall
    9 hours ago






  • 1




    $begingroup$
    Anyway, to potentially answer your question, your algebra moves are invalid if $x$ is negative, and there are solutions with negative $x$.
    $endgroup$
    – Randall
    9 hours ago






  • 1




    $begingroup$
    As motivation, you might consider whether the relation $y = arcsin(sin(x))$ is "algebraically equivalent" to $y=x$. (What about when $y=0$ and $x=pi$?) The same issue is at play here: $v = W(u)$ means that $u e^u = v$, but there is not always a unique $u$ with that property, and depending on which one you choose, your claim may or may not be valid.
    $endgroup$
    – Nate Eldredge
    8 hours ago














8












8








8


2



$begingroup$


Solving $x^y = y^x$ analytically in terms of the Lambert $W$ function



This "solution" for $x^y=y^x$ should simplify to $y=x$, but for some reason no pointed that out in the OP.



According to the stack exchange, the answer is $y= frac{-xW(-frac{ln(x)}{x})}{ln(x)}$. However, the term $frac{-ln(x)}{x}$ itself can be rewritten as



$$frac{-ln(x)}{x}=-ln(x)e^{-ln(x)}$$



Therefore, the productlog of that expression should simplify as follows,



$y= frac{-xW(-frac{ln(x)}{x})}{ln(x)}, $ $y= frac{-xW(-ln(x)e^{-ln(x)})}{ln(x)}, $ $y=frac{-x(-ln(x))}{ln(x)}=x$



Did this simplification just slip past everyone or is there something wrong about my algebra?










share|cite|improve this question











$endgroup$




Solving $x^y = y^x$ analytically in terms of the Lambert $W$ function



This "solution" for $x^y=y^x$ should simplify to $y=x$, but for some reason no pointed that out in the OP.



According to the stack exchange, the answer is $y= frac{-xW(-frac{ln(x)}{x})}{ln(x)}$. However, the term $frac{-ln(x)}{x}$ itself can be rewritten as



$$frac{-ln(x)}{x}=-ln(x)e^{-ln(x)}$$



Therefore, the productlog of that expression should simplify as follows,



$y= frac{-xW(-frac{ln(x)}{x})}{ln(x)}, $ $y= frac{-xW(-ln(x)e^{-ln(x)})}{ln(x)}, $ $y=frac{-x(-ln(x))}{ln(x)}=x$



Did this simplification just slip past everyone or is there something wrong about my algebra?







algebra-precalculus






share|cite|improve this question















share|cite|improve this question













share|cite|improve this question




share|cite|improve this question








edited 8 hours ago







user14554

















asked 9 hours ago









user14554user14554

435




435








  • 4




    $begingroup$
    Why should it reduce to that? $x=4$ and $y=2$ has $x neq y$.
    $endgroup$
    – Randall
    9 hours ago






  • 2




    $begingroup$
    No, $2^4=16=4^2$.
    $endgroup$
    – Randall
    9 hours ago






  • 5




    $begingroup$
    I'm just confused why the solution "should" simplify to $x=y$ when there are solutions that do not satisfy $x = y$.
    $endgroup$
    – Randall
    9 hours ago






  • 1




    $begingroup$
    Anyway, to potentially answer your question, your algebra moves are invalid if $x$ is negative, and there are solutions with negative $x$.
    $endgroup$
    – Randall
    9 hours ago






  • 1




    $begingroup$
    As motivation, you might consider whether the relation $y = arcsin(sin(x))$ is "algebraically equivalent" to $y=x$. (What about when $y=0$ and $x=pi$?) The same issue is at play here: $v = W(u)$ means that $u e^u = v$, but there is not always a unique $u$ with that property, and depending on which one you choose, your claim may or may not be valid.
    $endgroup$
    – Nate Eldredge
    8 hours ago














  • 4




    $begingroup$
    Why should it reduce to that? $x=4$ and $y=2$ has $x neq y$.
    $endgroup$
    – Randall
    9 hours ago






  • 2




    $begingroup$
    No, $2^4=16=4^2$.
    $endgroup$
    – Randall
    9 hours ago






  • 5




    $begingroup$
    I'm just confused why the solution "should" simplify to $x=y$ when there are solutions that do not satisfy $x = y$.
    $endgroup$
    – Randall
    9 hours ago






  • 1




    $begingroup$
    Anyway, to potentially answer your question, your algebra moves are invalid if $x$ is negative, and there are solutions with negative $x$.
    $endgroup$
    – Randall
    9 hours ago






  • 1




    $begingroup$
    As motivation, you might consider whether the relation $y = arcsin(sin(x))$ is "algebraically equivalent" to $y=x$. (What about when $y=0$ and $x=pi$?) The same issue is at play here: $v = W(u)$ means that $u e^u = v$, but there is not always a unique $u$ with that property, and depending on which one you choose, your claim may or may not be valid.
    $endgroup$
    – Nate Eldredge
    8 hours ago








4




4




$begingroup$
Why should it reduce to that? $x=4$ and $y=2$ has $x neq y$.
$endgroup$
– Randall
9 hours ago




$begingroup$
Why should it reduce to that? $x=4$ and $y=2$ has $x neq y$.
$endgroup$
– Randall
9 hours ago




2




2




$begingroup$
No, $2^4=16=4^2$.
$endgroup$
– Randall
9 hours ago




$begingroup$
No, $2^4=16=4^2$.
$endgroup$
– Randall
9 hours ago




5




5




$begingroup$
I'm just confused why the solution "should" simplify to $x=y$ when there are solutions that do not satisfy $x = y$.
$endgroup$
– Randall
9 hours ago




$begingroup$
I'm just confused why the solution "should" simplify to $x=y$ when there are solutions that do not satisfy $x = y$.
$endgroup$
– Randall
9 hours ago




1




1




$begingroup$
Anyway, to potentially answer your question, your algebra moves are invalid if $x$ is negative, and there are solutions with negative $x$.
$endgroup$
– Randall
9 hours ago




$begingroup$
Anyway, to potentially answer your question, your algebra moves are invalid if $x$ is negative, and there are solutions with negative $x$.
$endgroup$
– Randall
9 hours ago




1




1




$begingroup$
As motivation, you might consider whether the relation $y = arcsin(sin(x))$ is "algebraically equivalent" to $y=x$. (What about when $y=0$ and $x=pi$?) The same issue is at play here: $v = W(u)$ means that $u e^u = v$, but there is not always a unique $u$ with that property, and depending on which one you choose, your claim may or may not be valid.
$endgroup$
– Nate Eldredge
8 hours ago




$begingroup$
As motivation, you might consider whether the relation $y = arcsin(sin(x))$ is "algebraically equivalent" to $y=x$. (What about when $y=0$ and $x=pi$?) The same issue is at play here: $v = W(u)$ means that $u e^u = v$, but there is not always a unique $u$ with that property, and depending on which one you choose, your claim may or may not be valid.
$endgroup$
– Nate Eldredge
8 hours ago










2 Answers
2






active

oldest

votes


















9












$begingroup$

The Lambert $W$ function is not single-valued for negative arguments.



enter image description here



Using your "simplification" forces use of the lower branch, $W leq -1$ when you assume $W^{-1}(-ln x)$ only equals $-ln (x) mathrm{e}^{- ln x}$. (The same thing happens when you assume the only square root of $3^2$ is $3$ or the only arcsine of $1$ is $-3pi/2$.) You get two values from $W^{-1}(-ln x)$ having the same algebraic form, but one has $0 < x leq mathrm{e}$ and one has $x > mathrm{e}$. ("$3^2$" and "$(-3)^2$" have the same algebraic form, "$x^2$", but one has $x>0$ and one has $x < 0$.)



This is indicated explicitly in the identities at the Lambert $W$ function article on the English Wikipedia.



Edit: Got myself turned around with too many minus signs. I originally claimed the $x=y$ solutions were on $W geq -1$, but this is backwards. It is corrected above.






share|cite|improve this answer











$endgroup$













  • $begingroup$
    What is confusing is how $W(z)e^{W(z)}=z$ always simplifies no matter which branch you use, but $W(ze^{z})$ does not.
    $endgroup$
    – user14554
    8 hours ago












  • $begingroup$
    +1 for the first parenthetical.
    $endgroup$
    – Randall
    8 hours ago










  • $begingroup$
    @user14554 : This is the usual problem with inverse functions. $sqrt{9} = 3$, but "the things which square to $9$" is ${-3,3}$. This is always lurking around when you are solving equations.
    $endgroup$
    – Eric Towers
    8 hours ago








  • 1




    $begingroup$
    @user14554 : When $x = 6$, $W_{-1}$ gives $y = 6$ and $W_0$ gives $y = 1.624dots$. You get them back the same way you do with any other function whose domain must be restricted to obtain the inverse function: you use a full set of inverses whose ranges cover the entire domain of the unrestricted function.
    $endgroup$
    – Eric Towers
    8 hours ago








  • 1




    $begingroup$
    @user14554 : I disagree. Every time you apply a $W^{-1}$, you get a contribution from $W_0$ and another from $W_{-1}$. You are, of course, free to incorrectly ignore solutions. I, on the other hand, will continue to find that $(x^2 - 3)^2-1=0$ has four real roots.
    $endgroup$
    – Eric Towers
    7 hours ago



















1












$begingroup$

The solution is:



$$y = -frac{x Wleft(-frac{log (x)}{x}right)}{log (x)}$$



which has the following form:



enter image description here



Clearly there are solutions other than $x = y$. Indeed, we see that for $y=2$ we can have $x=2$ or $x=4$ (intersection between blue and red dashed line).






share|cite|improve this answer











$endgroup$













  • $begingroup$
    So it has something to do with the multiple branches of the log and productlog then. For $W_{0}(x)$ it simplifies, but when it changes to $W_{-1}(x)$ it doesn't.
    $endgroup$
    – user14554
    8 hours ago












  • $begingroup$
    I think OP's question is why isn't the blue line simply $y=x$? It is tantalizing that it is $y=x$ for a while and then there's a sudden change.
    $endgroup$
    – Randall
    8 hours ago












  • $begingroup$
    Right, why isn't it $y=x$ all the way.
    $endgroup$
    – user14554
    8 hours ago










  • $begingroup$
    @user14554 I see your question now.
    $endgroup$
    – Randall
    8 hours ago










  • $begingroup$
    user14554 and Randall: There must be a branch cut in the Lambert W function.
    $endgroup$
    – David G. Stork
    8 hours ago











Your Answer





StackExchange.ifUsing("editor", function () {
return StackExchange.using("mathjaxEditing", function () {
StackExchange.MarkdownEditor.creationCallbacks.add(function (editor, postfix) {
StackExchange.mathjaxEditing.prepareWmdForMathJax(editor, postfix, [["$", "$"], ["\\(","\\)"]]);
});
});
}, "mathjax-editing");

StackExchange.ready(function() {
var channelOptions = {
tags: "".split(" "),
id: "69"
};
initTagRenderer("".split(" "), "".split(" "), channelOptions);

StackExchange.using("externalEditor", function() {
// Have to fire editor after snippets, if snippets enabled
if (StackExchange.settings.snippets.snippetsEnabled) {
StackExchange.using("snippets", function() {
createEditor();
});
}
else {
createEditor();
}
});

function createEditor() {
StackExchange.prepareEditor({
heartbeatType: 'answer',
autoActivateHeartbeat: false,
convertImagesToLinks: true,
noModals: true,
showLowRepImageUploadWarning: true,
reputationToPostImages: 10,
bindNavPrevention: true,
postfix: "",
imageUploader: {
brandingHtml: "Powered by u003ca class="icon-imgur-white" href="https://imgur.com/"u003eu003c/au003e",
contentPolicyHtml: "User contributions licensed under u003ca href="https://creativecommons.org/licenses/by-sa/3.0/"u003ecc by-sa 3.0 with attribution requiredu003c/au003e u003ca href="https://stackoverflow.com/legal/content-policy"u003e(content policy)u003c/au003e",
allowUrls: true
},
noCode: true, onDemand: true,
discardSelector: ".discard-answer"
,immediatelyShowMarkdownHelp:true
});


}
});














draft saved

draft discarded


















StackExchange.ready(
function () {
StackExchange.openid.initPostLogin('.new-post-login', 'https%3a%2f%2fmath.stackexchange.com%2fquestions%2f3072828%2fwhy-didnt-they-simplify-this%23new-answer', 'question_page');
}
);

Post as a guest















Required, but never shown

























2 Answers
2






active

oldest

votes








2 Answers
2






active

oldest

votes









active

oldest

votes






active

oldest

votes









9












$begingroup$

The Lambert $W$ function is not single-valued for negative arguments.



enter image description here



Using your "simplification" forces use of the lower branch, $W leq -1$ when you assume $W^{-1}(-ln x)$ only equals $-ln (x) mathrm{e}^{- ln x}$. (The same thing happens when you assume the only square root of $3^2$ is $3$ or the only arcsine of $1$ is $-3pi/2$.) You get two values from $W^{-1}(-ln x)$ having the same algebraic form, but one has $0 < x leq mathrm{e}$ and one has $x > mathrm{e}$. ("$3^2$" and "$(-3)^2$" have the same algebraic form, "$x^2$", but one has $x>0$ and one has $x < 0$.)



This is indicated explicitly in the identities at the Lambert $W$ function article on the English Wikipedia.



Edit: Got myself turned around with too many minus signs. I originally claimed the $x=y$ solutions were on $W geq -1$, but this is backwards. It is corrected above.






share|cite|improve this answer











$endgroup$













  • $begingroup$
    What is confusing is how $W(z)e^{W(z)}=z$ always simplifies no matter which branch you use, but $W(ze^{z})$ does not.
    $endgroup$
    – user14554
    8 hours ago












  • $begingroup$
    +1 for the first parenthetical.
    $endgroup$
    – Randall
    8 hours ago










  • $begingroup$
    @user14554 : This is the usual problem with inverse functions. $sqrt{9} = 3$, but "the things which square to $9$" is ${-3,3}$. This is always lurking around when you are solving equations.
    $endgroup$
    – Eric Towers
    8 hours ago








  • 1




    $begingroup$
    @user14554 : When $x = 6$, $W_{-1}$ gives $y = 6$ and $W_0$ gives $y = 1.624dots$. You get them back the same way you do with any other function whose domain must be restricted to obtain the inverse function: you use a full set of inverses whose ranges cover the entire domain of the unrestricted function.
    $endgroup$
    – Eric Towers
    8 hours ago








  • 1




    $begingroup$
    @user14554 : I disagree. Every time you apply a $W^{-1}$, you get a contribution from $W_0$ and another from $W_{-1}$. You are, of course, free to incorrectly ignore solutions. I, on the other hand, will continue to find that $(x^2 - 3)^2-1=0$ has four real roots.
    $endgroup$
    – Eric Towers
    7 hours ago
















9












$begingroup$

The Lambert $W$ function is not single-valued for negative arguments.



enter image description here



Using your "simplification" forces use of the lower branch, $W leq -1$ when you assume $W^{-1}(-ln x)$ only equals $-ln (x) mathrm{e}^{- ln x}$. (The same thing happens when you assume the only square root of $3^2$ is $3$ or the only arcsine of $1$ is $-3pi/2$.) You get two values from $W^{-1}(-ln x)$ having the same algebraic form, but one has $0 < x leq mathrm{e}$ and one has $x > mathrm{e}$. ("$3^2$" and "$(-3)^2$" have the same algebraic form, "$x^2$", but one has $x>0$ and one has $x < 0$.)



This is indicated explicitly in the identities at the Lambert $W$ function article on the English Wikipedia.



Edit: Got myself turned around with too many minus signs. I originally claimed the $x=y$ solutions were on $W geq -1$, but this is backwards. It is corrected above.






share|cite|improve this answer











$endgroup$













  • $begingroup$
    What is confusing is how $W(z)e^{W(z)}=z$ always simplifies no matter which branch you use, but $W(ze^{z})$ does not.
    $endgroup$
    – user14554
    8 hours ago












  • $begingroup$
    +1 for the first parenthetical.
    $endgroup$
    – Randall
    8 hours ago










  • $begingroup$
    @user14554 : This is the usual problem with inverse functions. $sqrt{9} = 3$, but "the things which square to $9$" is ${-3,3}$. This is always lurking around when you are solving equations.
    $endgroup$
    – Eric Towers
    8 hours ago








  • 1




    $begingroup$
    @user14554 : When $x = 6$, $W_{-1}$ gives $y = 6$ and $W_0$ gives $y = 1.624dots$. You get them back the same way you do with any other function whose domain must be restricted to obtain the inverse function: you use a full set of inverses whose ranges cover the entire domain of the unrestricted function.
    $endgroup$
    – Eric Towers
    8 hours ago








  • 1




    $begingroup$
    @user14554 : I disagree. Every time you apply a $W^{-1}$, you get a contribution from $W_0$ and another from $W_{-1}$. You are, of course, free to incorrectly ignore solutions. I, on the other hand, will continue to find that $(x^2 - 3)^2-1=0$ has four real roots.
    $endgroup$
    – Eric Towers
    7 hours ago














9












9








9





$begingroup$

The Lambert $W$ function is not single-valued for negative arguments.



enter image description here



Using your "simplification" forces use of the lower branch, $W leq -1$ when you assume $W^{-1}(-ln x)$ only equals $-ln (x) mathrm{e}^{- ln x}$. (The same thing happens when you assume the only square root of $3^2$ is $3$ or the only arcsine of $1$ is $-3pi/2$.) You get two values from $W^{-1}(-ln x)$ having the same algebraic form, but one has $0 < x leq mathrm{e}$ and one has $x > mathrm{e}$. ("$3^2$" and "$(-3)^2$" have the same algebraic form, "$x^2$", but one has $x>0$ and one has $x < 0$.)



This is indicated explicitly in the identities at the Lambert $W$ function article on the English Wikipedia.



Edit: Got myself turned around with too many minus signs. I originally claimed the $x=y$ solutions were on $W geq -1$, but this is backwards. It is corrected above.






share|cite|improve this answer











$endgroup$



The Lambert $W$ function is not single-valued for negative arguments.



enter image description here



Using your "simplification" forces use of the lower branch, $W leq -1$ when you assume $W^{-1}(-ln x)$ only equals $-ln (x) mathrm{e}^{- ln x}$. (The same thing happens when you assume the only square root of $3^2$ is $3$ or the only arcsine of $1$ is $-3pi/2$.) You get two values from $W^{-1}(-ln x)$ having the same algebraic form, but one has $0 < x leq mathrm{e}$ and one has $x > mathrm{e}$. ("$3^2$" and "$(-3)^2$" have the same algebraic form, "$x^2$", but one has $x>0$ and one has $x < 0$.)



This is indicated explicitly in the identities at the Lambert $W$ function article on the English Wikipedia.



Edit: Got myself turned around with too many minus signs. I originally claimed the $x=y$ solutions were on $W geq -1$, but this is backwards. It is corrected above.







share|cite|improve this answer














share|cite|improve this answer



share|cite|improve this answer








edited 8 hours ago

























answered 8 hours ago









Eric TowersEric Towers

32.3k22267




32.3k22267












  • $begingroup$
    What is confusing is how $W(z)e^{W(z)}=z$ always simplifies no matter which branch you use, but $W(ze^{z})$ does not.
    $endgroup$
    – user14554
    8 hours ago












  • $begingroup$
    +1 for the first parenthetical.
    $endgroup$
    – Randall
    8 hours ago










  • $begingroup$
    @user14554 : This is the usual problem with inverse functions. $sqrt{9} = 3$, but "the things which square to $9$" is ${-3,3}$. This is always lurking around when you are solving equations.
    $endgroup$
    – Eric Towers
    8 hours ago








  • 1




    $begingroup$
    @user14554 : When $x = 6$, $W_{-1}$ gives $y = 6$ and $W_0$ gives $y = 1.624dots$. You get them back the same way you do with any other function whose domain must be restricted to obtain the inverse function: you use a full set of inverses whose ranges cover the entire domain of the unrestricted function.
    $endgroup$
    – Eric Towers
    8 hours ago








  • 1




    $begingroup$
    @user14554 : I disagree. Every time you apply a $W^{-1}$, you get a contribution from $W_0$ and another from $W_{-1}$. You are, of course, free to incorrectly ignore solutions. I, on the other hand, will continue to find that $(x^2 - 3)^2-1=0$ has four real roots.
    $endgroup$
    – Eric Towers
    7 hours ago


















  • $begingroup$
    What is confusing is how $W(z)e^{W(z)}=z$ always simplifies no matter which branch you use, but $W(ze^{z})$ does not.
    $endgroup$
    – user14554
    8 hours ago












  • $begingroup$
    +1 for the first parenthetical.
    $endgroup$
    – Randall
    8 hours ago










  • $begingroup$
    @user14554 : This is the usual problem with inverse functions. $sqrt{9} = 3$, but "the things which square to $9$" is ${-3,3}$. This is always lurking around when you are solving equations.
    $endgroup$
    – Eric Towers
    8 hours ago








  • 1




    $begingroup$
    @user14554 : When $x = 6$, $W_{-1}$ gives $y = 6$ and $W_0$ gives $y = 1.624dots$. You get them back the same way you do with any other function whose domain must be restricted to obtain the inverse function: you use a full set of inverses whose ranges cover the entire domain of the unrestricted function.
    $endgroup$
    – Eric Towers
    8 hours ago








  • 1




    $begingroup$
    @user14554 : I disagree. Every time you apply a $W^{-1}$, you get a contribution from $W_0$ and another from $W_{-1}$. You are, of course, free to incorrectly ignore solutions. I, on the other hand, will continue to find that $(x^2 - 3)^2-1=0$ has four real roots.
    $endgroup$
    – Eric Towers
    7 hours ago
















$begingroup$
What is confusing is how $W(z)e^{W(z)}=z$ always simplifies no matter which branch you use, but $W(ze^{z})$ does not.
$endgroup$
– user14554
8 hours ago






$begingroup$
What is confusing is how $W(z)e^{W(z)}=z$ always simplifies no matter which branch you use, but $W(ze^{z})$ does not.
$endgroup$
– user14554
8 hours ago














$begingroup$
+1 for the first parenthetical.
$endgroup$
– Randall
8 hours ago




$begingroup$
+1 for the first parenthetical.
$endgroup$
– Randall
8 hours ago












$begingroup$
@user14554 : This is the usual problem with inverse functions. $sqrt{9} = 3$, but "the things which square to $9$" is ${-3,3}$. This is always lurking around when you are solving equations.
$endgroup$
– Eric Towers
8 hours ago






$begingroup$
@user14554 : This is the usual problem with inverse functions. $sqrt{9} = 3$, but "the things which square to $9$" is ${-3,3}$. This is always lurking around when you are solving equations.
$endgroup$
– Eric Towers
8 hours ago






1




1




$begingroup$
@user14554 : When $x = 6$, $W_{-1}$ gives $y = 6$ and $W_0$ gives $y = 1.624dots$. You get them back the same way you do with any other function whose domain must be restricted to obtain the inverse function: you use a full set of inverses whose ranges cover the entire domain of the unrestricted function.
$endgroup$
– Eric Towers
8 hours ago






$begingroup$
@user14554 : When $x = 6$, $W_{-1}$ gives $y = 6$ and $W_0$ gives $y = 1.624dots$. You get them back the same way you do with any other function whose domain must be restricted to obtain the inverse function: you use a full set of inverses whose ranges cover the entire domain of the unrestricted function.
$endgroup$
– Eric Towers
8 hours ago






1




1




$begingroup$
@user14554 : I disagree. Every time you apply a $W^{-1}$, you get a contribution from $W_0$ and another from $W_{-1}$. You are, of course, free to incorrectly ignore solutions. I, on the other hand, will continue to find that $(x^2 - 3)^2-1=0$ has four real roots.
$endgroup$
– Eric Towers
7 hours ago




$begingroup$
@user14554 : I disagree. Every time you apply a $W^{-1}$, you get a contribution from $W_0$ and another from $W_{-1}$. You are, of course, free to incorrectly ignore solutions. I, on the other hand, will continue to find that $(x^2 - 3)^2-1=0$ has four real roots.
$endgroup$
– Eric Towers
7 hours ago











1












$begingroup$

The solution is:



$$y = -frac{x Wleft(-frac{log (x)}{x}right)}{log (x)}$$



which has the following form:



enter image description here



Clearly there are solutions other than $x = y$. Indeed, we see that for $y=2$ we can have $x=2$ or $x=4$ (intersection between blue and red dashed line).






share|cite|improve this answer











$endgroup$













  • $begingroup$
    So it has something to do with the multiple branches of the log and productlog then. For $W_{0}(x)$ it simplifies, but when it changes to $W_{-1}(x)$ it doesn't.
    $endgroup$
    – user14554
    8 hours ago












  • $begingroup$
    I think OP's question is why isn't the blue line simply $y=x$? It is tantalizing that it is $y=x$ for a while and then there's a sudden change.
    $endgroup$
    – Randall
    8 hours ago












  • $begingroup$
    Right, why isn't it $y=x$ all the way.
    $endgroup$
    – user14554
    8 hours ago










  • $begingroup$
    @user14554 I see your question now.
    $endgroup$
    – Randall
    8 hours ago










  • $begingroup$
    user14554 and Randall: There must be a branch cut in the Lambert W function.
    $endgroup$
    – David G. Stork
    8 hours ago
















1












$begingroup$

The solution is:



$$y = -frac{x Wleft(-frac{log (x)}{x}right)}{log (x)}$$



which has the following form:



enter image description here



Clearly there are solutions other than $x = y$. Indeed, we see that for $y=2$ we can have $x=2$ or $x=4$ (intersection between blue and red dashed line).






share|cite|improve this answer











$endgroup$













  • $begingroup$
    So it has something to do with the multiple branches of the log and productlog then. For $W_{0}(x)$ it simplifies, but when it changes to $W_{-1}(x)$ it doesn't.
    $endgroup$
    – user14554
    8 hours ago












  • $begingroup$
    I think OP's question is why isn't the blue line simply $y=x$? It is tantalizing that it is $y=x$ for a while and then there's a sudden change.
    $endgroup$
    – Randall
    8 hours ago












  • $begingroup$
    Right, why isn't it $y=x$ all the way.
    $endgroup$
    – user14554
    8 hours ago










  • $begingroup$
    @user14554 I see your question now.
    $endgroup$
    – Randall
    8 hours ago










  • $begingroup$
    user14554 and Randall: There must be a branch cut in the Lambert W function.
    $endgroup$
    – David G. Stork
    8 hours ago














1












1








1





$begingroup$

The solution is:



$$y = -frac{x Wleft(-frac{log (x)}{x}right)}{log (x)}$$



which has the following form:



enter image description here



Clearly there are solutions other than $x = y$. Indeed, we see that for $y=2$ we can have $x=2$ or $x=4$ (intersection between blue and red dashed line).






share|cite|improve this answer











$endgroup$



The solution is:



$$y = -frac{x Wleft(-frac{log (x)}{x}right)}{log (x)}$$



which has the following form:



enter image description here



Clearly there are solutions other than $x = y$. Indeed, we see that for $y=2$ we can have $x=2$ or $x=4$ (intersection between blue and red dashed line).







share|cite|improve this answer














share|cite|improve this answer



share|cite|improve this answer








edited 8 hours ago

























answered 8 hours ago









David G. StorkDavid G. Stork

10.3k21332




10.3k21332












  • $begingroup$
    So it has something to do with the multiple branches of the log and productlog then. For $W_{0}(x)$ it simplifies, but when it changes to $W_{-1}(x)$ it doesn't.
    $endgroup$
    – user14554
    8 hours ago












  • $begingroup$
    I think OP's question is why isn't the blue line simply $y=x$? It is tantalizing that it is $y=x$ for a while and then there's a sudden change.
    $endgroup$
    – Randall
    8 hours ago












  • $begingroup$
    Right, why isn't it $y=x$ all the way.
    $endgroup$
    – user14554
    8 hours ago










  • $begingroup$
    @user14554 I see your question now.
    $endgroup$
    – Randall
    8 hours ago










  • $begingroup$
    user14554 and Randall: There must be a branch cut in the Lambert W function.
    $endgroup$
    – David G. Stork
    8 hours ago


















  • $begingroup$
    So it has something to do with the multiple branches of the log and productlog then. For $W_{0}(x)$ it simplifies, but when it changes to $W_{-1}(x)$ it doesn't.
    $endgroup$
    – user14554
    8 hours ago












  • $begingroup$
    I think OP's question is why isn't the blue line simply $y=x$? It is tantalizing that it is $y=x$ for a while and then there's a sudden change.
    $endgroup$
    – Randall
    8 hours ago












  • $begingroup$
    Right, why isn't it $y=x$ all the way.
    $endgroup$
    – user14554
    8 hours ago










  • $begingroup$
    @user14554 I see your question now.
    $endgroup$
    – Randall
    8 hours ago










  • $begingroup$
    user14554 and Randall: There must be a branch cut in the Lambert W function.
    $endgroup$
    – David G. Stork
    8 hours ago
















$begingroup$
So it has something to do with the multiple branches of the log and productlog then. For $W_{0}(x)$ it simplifies, but when it changes to $W_{-1}(x)$ it doesn't.
$endgroup$
– user14554
8 hours ago






$begingroup$
So it has something to do with the multiple branches of the log and productlog then. For $W_{0}(x)$ it simplifies, but when it changes to $W_{-1}(x)$ it doesn't.
$endgroup$
– user14554
8 hours ago














$begingroup$
I think OP's question is why isn't the blue line simply $y=x$? It is tantalizing that it is $y=x$ for a while and then there's a sudden change.
$endgroup$
– Randall
8 hours ago






$begingroup$
I think OP's question is why isn't the blue line simply $y=x$? It is tantalizing that it is $y=x$ for a while and then there's a sudden change.
$endgroup$
– Randall
8 hours ago














$begingroup$
Right, why isn't it $y=x$ all the way.
$endgroup$
– user14554
8 hours ago




$begingroup$
Right, why isn't it $y=x$ all the way.
$endgroup$
– user14554
8 hours ago












$begingroup$
@user14554 I see your question now.
$endgroup$
– Randall
8 hours ago




$begingroup$
@user14554 I see your question now.
$endgroup$
– Randall
8 hours ago












$begingroup$
user14554 and Randall: There must be a branch cut in the Lambert W function.
$endgroup$
– David G. Stork
8 hours ago




$begingroup$
user14554 and Randall: There must be a branch cut in the Lambert W function.
$endgroup$
– David G. Stork
8 hours ago


















draft saved

draft discarded




















































Thanks for contributing an answer to Mathematics Stack Exchange!


  • Please be sure to answer the question. Provide details and share your research!

But avoid



  • Asking for help, clarification, or responding to other answers.

  • Making statements based on opinion; back them up with references or personal experience.


Use MathJax to format equations. MathJax reference.


To learn more, see our tips on writing great answers.




draft saved


draft discarded














StackExchange.ready(
function () {
StackExchange.openid.initPostLogin('.new-post-login', 'https%3a%2f%2fmath.stackexchange.com%2fquestions%2f3072828%2fwhy-didnt-they-simplify-this%23new-answer', 'question_page');
}
);

Post as a guest















Required, but never shown





















































Required, but never shown














Required, but never shown












Required, but never shown







Required, but never shown

































Required, but never shown














Required, but never shown












Required, but never shown







Required, but never shown







Popular posts from this blog

GameSpot

connect to host localhost port 22: Connection refused

Getting a Wifi WPA2 wifi connection